Last visit was: 26 Apr 2024, 00:43 It is currently 26 Apr 2024, 00:43

Close

GRE Prep Club Daily Prep

Thank you for using the timer - this advanced tool can estimate your performance and suggest more practice questions. We have subscribed you to Daily Prep Questions via email.

Customized
for You

we will pick new questions that match your level based on your Timer History

Track
Your Progress

every week, we’ll send you an estimated GRE score based on your performance

Practice
Pays

we will pick new questions that match your level based on your Timer History

Not interested in getting valuable practice questions and articles delivered to your email? No problem, unsubscribe here.

Close

Request Expert Reply

Confirm Cancel
SORT BY:
Date
GRE Prep Club Team Member
Joined: 20 Feb 2017
Posts: 2506
Own Kudos [?]: 3249 [1]
Given Kudos: 1051
GPA: 3.39
Send PM
Manager
Manager
Joined: 18 Jan 2022
Posts: 69
Own Kudos [?]: 51 [0]
Given Kudos: 144
Send PM
Manager
Manager
Joined: 06 Jun 2021
Posts: 78
Own Kudos [?]: 8 [0]
Given Kudos: 61
Send PM
Verbal Expert
Joined: 18 Apr 2015
Posts: 28635
Own Kudos [?]: 33129 [1]
Given Kudos: 25177
Send PM
Re: While there is no blueprint for transforming a largely government-cont [#permalink]
1
Expert Reply
kk117 wrote:
For question 7, doesn't D seem more suitable?

My thinking behind choosing D:

We have a controversial viewpoint "Some economists have suggested that giving away free shares would provide a needed acceleration of the privatization process."

However, the paradox here is that "if shares are obtained for free ( instead of being bought with employees' money ), they wouldn't necessarily contribute to the privatization process".

I think both options A and option D are valid.



(A) counter a position that the author of the passage believes is incorrect
(B) state a solution to a problem described in the previous sentence
(C) show how opponents of the viewpoint of the author of the passage have supported their arguments
(D) point out a paradox contained in a controversial viewpoint
(E) present a historical maxim to challenge the principle introduced in the third paragraph
Logical structure
Looking at the quotation’s context leads to an understanding of why the quotation was used. Paine’ s quotation offers a concise and time-honored counterargument to the view voiced in the preceding sentence.The economists suggest giving away free shares,but the author notes that these economists are forgetting that,according to Paine,people do not value what they get too cheaply.The author uses the quotation to show the basic error in the economists’ thinking.
A Correct.The author uses Paine's quotation as an apt counter to the economists’ suggestion.
B The quotation attacks the solution posed in the previous sentence.
C The author agrees with Paine,as is evident in the final 1ines of the passage.
D The author implies that a viewpoint is ill advised.but does not say it is controversial.
E Paine's maxim does not challenge the principle of self-interest.
The correct answer is A.
Prep Club for GRE Bot
[#permalink]
Moderators:
GRE Instructor
218 posts
GRE Instructor
1029 posts

Powered by phpBB © phpBB Group | Emoji artwork provided by EmojiOne